The point of inflection on the curve $𝑦=𝑥^3−𝑎𝑥^2−𝑏𝑥+𝑐$ is a stationary point of...

Could solar power be utilized and substitute coal in the 19th Century

Should I stop contributing to retirement accounts?

Can a significant change in incentives void an employment contract?

When quoting, must I also copy hyphens used to divide words that continue on the next line?

How should I respond when I lied about my education and the company finds out through background check?

Translation of Scottish 16th century church stained glass

Fly on a jet pack vs fly with a jet pack?

Can I sign legal documents with a smiley face?

Is camera lens focus an exact point or a range?

Why did the EU agree to delay the Brexit deadline?

Open a doc from terminal, but not by its name

Freedom of speech and where it applies

How can Trident be so inexpensive? Will it orbit Triton or just do a (slow) flyby?

Has Darkwing Duck ever met Scrooge McDuck?

How do I extrude a face to a single vertex

How do ground effect vehicles perform turns?

How to color a curve

Do Legal Documents Require Signing In Standard Pen Colors?

THT: What is a squared annular “ring”?

Why did the HMS Bounty go back to a time when whales are already rare?

What (else) happened July 1st 1858 in London?

Indicating multiple different modes of speech (fantasy language or telepathy)

Is it possible to have a strip of cold climate in the middle of a planet?

Are lightweight LN wallets vulnerable to transaction withholding?



The point of inflection on the curve $𝑦=𝑥^3−𝑎𝑥^2−𝑏𝑥+𝑐$ is a stationary point of inflexion. Show that $b=8a^2$. [duplicate]


Show that $b=8a^2$Find the coordinates of any stationary points on the curve $y = {1 over {1 + {x^2}}}$ and state it's natureStrictly monotone real function: stationary point, non-differentiable pointIs $t=0$ a stationary point?The second and third derivitive tests give unexpected $0$How do I show that something only has one stationary point?Sufficient condition for inflection pointThis second derivative is showing a point of inflection rather than a minimum pointa point of inflectionNormal to a curve at the point x=1Show that if the curve $y = f(x)$ has a maximum stationary point at $x = a$













0












$begingroup$



This question already has an answer here:




  • Show that $b=8a^2$

    2 answers




The point of inflection on the curve $𝑦=𝑥^3−𝑎𝑥^2−𝑏𝑥+𝑐$ is a stationary point of inflection. Show that $b=8a2$.



Thank you for your help.



Edit: The solution to this question drafted in another post is wrong, according to the book. That is why I post it again, in a separate post.



Edit 2: I am not sure, then, why some of you still mark it as a duplicate.










share|cite|improve this question











$endgroup$



marked as duplicate by Dietrich Burde, RRL, Saad, Parcly Taxel, John Omielan Mar 14 at 14:54


This question has been asked before and already has an answer. If those answers do not fully address your question, please ask a new question.


















  • $begingroup$
    The thing is that in that post the solution does not show that this is actually the case - that b = 8a2; and such, according to the solution in the book, is wrong.
    $endgroup$
    – Marcin
    Mar 14 at 12:39










  • $begingroup$
    Welcome to Math Stack Exchange. Please use MathJax
    $endgroup$
    – J. W. Tanner
    Mar 14 at 13:20


















0












$begingroup$



This question already has an answer here:




  • Show that $b=8a^2$

    2 answers




The point of inflection on the curve $𝑦=𝑥^3−𝑎𝑥^2−𝑏𝑥+𝑐$ is a stationary point of inflection. Show that $b=8a2$.



Thank you for your help.



Edit: The solution to this question drafted in another post is wrong, according to the book. That is why I post it again, in a separate post.



Edit 2: I am not sure, then, why some of you still mark it as a duplicate.










share|cite|improve this question











$endgroup$



marked as duplicate by Dietrich Burde, RRL, Saad, Parcly Taxel, John Omielan Mar 14 at 14:54


This question has been asked before and already has an answer. If those answers do not fully address your question, please ask a new question.


















  • $begingroup$
    The thing is that in that post the solution does not show that this is actually the case - that b = 8a2; and such, according to the solution in the book, is wrong.
    $endgroup$
    – Marcin
    Mar 14 at 12:39










  • $begingroup$
    Welcome to Math Stack Exchange. Please use MathJax
    $endgroup$
    – J. W. Tanner
    Mar 14 at 13:20
















0












0








0


1



$begingroup$



This question already has an answer here:




  • Show that $b=8a^2$

    2 answers




The point of inflection on the curve $𝑦=𝑥^3−𝑎𝑥^2−𝑏𝑥+𝑐$ is a stationary point of inflection. Show that $b=8a2$.



Thank you for your help.



Edit: The solution to this question drafted in another post is wrong, according to the book. That is why I post it again, in a separate post.



Edit 2: I am not sure, then, why some of you still mark it as a duplicate.










share|cite|improve this question











$endgroup$





This question already has an answer here:




  • Show that $b=8a^2$

    2 answers




The point of inflection on the curve $𝑦=𝑥^3−𝑎𝑥^2−𝑏𝑥+𝑐$ is a stationary point of inflection. Show that $b=8a2$.



Thank you for your help.



Edit: The solution to this question drafted in another post is wrong, according to the book. That is why I post it again, in a separate post.



Edit 2: I am not sure, then, why some of you still mark it as a duplicate.





This question already has an answer here:




  • Show that $b=8a^2$

    2 answers








calculus derivatives






share|cite|improve this question















share|cite|improve this question













share|cite|improve this question




share|cite|improve this question








edited Mar 17 at 16:57







Marcin

















asked Mar 14 at 12:19









MarcinMarcin

83




83




marked as duplicate by Dietrich Burde, RRL, Saad, Parcly Taxel, John Omielan Mar 14 at 14:54


This question has been asked before and already has an answer. If those answers do not fully address your question, please ask a new question.









marked as duplicate by Dietrich Burde, RRL, Saad, Parcly Taxel, John Omielan Mar 14 at 14:54


This question has been asked before and already has an answer. If those answers do not fully address your question, please ask a new question.














  • $begingroup$
    The thing is that in that post the solution does not show that this is actually the case - that b = 8a2; and such, according to the solution in the book, is wrong.
    $endgroup$
    – Marcin
    Mar 14 at 12:39










  • $begingroup$
    Welcome to Math Stack Exchange. Please use MathJax
    $endgroup$
    – J. W. Tanner
    Mar 14 at 13:20




















  • $begingroup$
    The thing is that in that post the solution does not show that this is actually the case - that b = 8a2; and such, according to the solution in the book, is wrong.
    $endgroup$
    – Marcin
    Mar 14 at 12:39










  • $begingroup$
    Welcome to Math Stack Exchange. Please use MathJax
    $endgroup$
    – J. W. Tanner
    Mar 14 at 13:20


















$begingroup$
The thing is that in that post the solution does not show that this is actually the case - that b = 8a2; and such, according to the solution in the book, is wrong.
$endgroup$
– Marcin
Mar 14 at 12:39




$begingroup$
The thing is that in that post the solution does not show that this is actually the case - that b = 8a2; and such, according to the solution in the book, is wrong.
$endgroup$
– Marcin
Mar 14 at 12:39












$begingroup$
Welcome to Math Stack Exchange. Please use MathJax
$endgroup$
– J. W. Tanner
Mar 14 at 13:20






$begingroup$
Welcome to Math Stack Exchange. Please use MathJax
$endgroup$
– J. W. Tanner
Mar 14 at 13:20












1 Answer
1






active

oldest

votes


















0












$begingroup$

To find stationary points we need $f'(x)=0$ and to find inflection points we need to find where $f''(x)=0$.



$$f(x)=x^3-ax^2-bx+c$$
$$f'(x)=3x^2-2ax-b$$
$$f''(x)=6x-2a$$



So to find the inflection point we need $$6x-2a=0$$ $$6x=2a$$ $$x=frac{a}{3}.$$



In order for this point to be stationary we need $f'(frac{a}{3})=0$.



$$3bigg(frac{a}{3}bigg)^2-2abigg(frac{a}{3}bigg)-b=0$$
$$frac{a^2}{3}-frac{2a^2}{3}-b=0$$



Now you should be able to plug in $a$ and $b$ and show that this equation is correct. Is your question saying that $a=2$ and $b=8$? Or $b=8a^2$? In either case, this inflection point would not be a stationary point in this function.






share|cite|improve this answer









$endgroup$













  • $begingroup$
    The latter one. It looks like there is a mistake in the answers, then. Thank you very much for solving it.
    $endgroup$
    – Marcin
    Mar 14 at 14:24










  • $begingroup$
    You’re welcome, glad I could help.
    $endgroup$
    – Jake O
    Mar 14 at 14:42


















1 Answer
1






active

oldest

votes








1 Answer
1






active

oldest

votes









active

oldest

votes






active

oldest

votes









0












$begingroup$

To find stationary points we need $f'(x)=0$ and to find inflection points we need to find where $f''(x)=0$.



$$f(x)=x^3-ax^2-bx+c$$
$$f'(x)=3x^2-2ax-b$$
$$f''(x)=6x-2a$$



So to find the inflection point we need $$6x-2a=0$$ $$6x=2a$$ $$x=frac{a}{3}.$$



In order for this point to be stationary we need $f'(frac{a}{3})=0$.



$$3bigg(frac{a}{3}bigg)^2-2abigg(frac{a}{3}bigg)-b=0$$
$$frac{a^2}{3}-frac{2a^2}{3}-b=0$$



Now you should be able to plug in $a$ and $b$ and show that this equation is correct. Is your question saying that $a=2$ and $b=8$? Or $b=8a^2$? In either case, this inflection point would not be a stationary point in this function.






share|cite|improve this answer









$endgroup$













  • $begingroup$
    The latter one. It looks like there is a mistake in the answers, then. Thank you very much for solving it.
    $endgroup$
    – Marcin
    Mar 14 at 14:24










  • $begingroup$
    You’re welcome, glad I could help.
    $endgroup$
    – Jake O
    Mar 14 at 14:42
















0












$begingroup$

To find stationary points we need $f'(x)=0$ and to find inflection points we need to find where $f''(x)=0$.



$$f(x)=x^3-ax^2-bx+c$$
$$f'(x)=3x^2-2ax-b$$
$$f''(x)=6x-2a$$



So to find the inflection point we need $$6x-2a=0$$ $$6x=2a$$ $$x=frac{a}{3}.$$



In order for this point to be stationary we need $f'(frac{a}{3})=0$.



$$3bigg(frac{a}{3}bigg)^2-2abigg(frac{a}{3}bigg)-b=0$$
$$frac{a^2}{3}-frac{2a^2}{3}-b=0$$



Now you should be able to plug in $a$ and $b$ and show that this equation is correct. Is your question saying that $a=2$ and $b=8$? Or $b=8a^2$? In either case, this inflection point would not be a stationary point in this function.






share|cite|improve this answer









$endgroup$













  • $begingroup$
    The latter one. It looks like there is a mistake in the answers, then. Thank you very much for solving it.
    $endgroup$
    – Marcin
    Mar 14 at 14:24










  • $begingroup$
    You’re welcome, glad I could help.
    $endgroup$
    – Jake O
    Mar 14 at 14:42














0












0








0





$begingroup$

To find stationary points we need $f'(x)=0$ and to find inflection points we need to find where $f''(x)=0$.



$$f(x)=x^3-ax^2-bx+c$$
$$f'(x)=3x^2-2ax-b$$
$$f''(x)=6x-2a$$



So to find the inflection point we need $$6x-2a=0$$ $$6x=2a$$ $$x=frac{a}{3}.$$



In order for this point to be stationary we need $f'(frac{a}{3})=0$.



$$3bigg(frac{a}{3}bigg)^2-2abigg(frac{a}{3}bigg)-b=0$$
$$frac{a^2}{3}-frac{2a^2}{3}-b=0$$



Now you should be able to plug in $a$ and $b$ and show that this equation is correct. Is your question saying that $a=2$ and $b=8$? Or $b=8a^2$? In either case, this inflection point would not be a stationary point in this function.






share|cite|improve this answer









$endgroup$



To find stationary points we need $f'(x)=0$ and to find inflection points we need to find where $f''(x)=0$.



$$f(x)=x^3-ax^2-bx+c$$
$$f'(x)=3x^2-2ax-b$$
$$f''(x)=6x-2a$$



So to find the inflection point we need $$6x-2a=0$$ $$6x=2a$$ $$x=frac{a}{3}.$$



In order for this point to be stationary we need $f'(frac{a}{3})=0$.



$$3bigg(frac{a}{3}bigg)^2-2abigg(frac{a}{3}bigg)-b=0$$
$$frac{a^2}{3}-frac{2a^2}{3}-b=0$$



Now you should be able to plug in $a$ and $b$ and show that this equation is correct. Is your question saying that $a=2$ and $b=8$? Or $b=8a^2$? In either case, this inflection point would not be a stationary point in this function.







share|cite|improve this answer












share|cite|improve this answer



share|cite|improve this answer










answered Mar 14 at 14:16









Jake OJake O

1665




1665












  • $begingroup$
    The latter one. It looks like there is a mistake in the answers, then. Thank you very much for solving it.
    $endgroup$
    – Marcin
    Mar 14 at 14:24










  • $begingroup$
    You’re welcome, glad I could help.
    $endgroup$
    – Jake O
    Mar 14 at 14:42


















  • $begingroup$
    The latter one. It looks like there is a mistake in the answers, then. Thank you very much for solving it.
    $endgroup$
    – Marcin
    Mar 14 at 14:24










  • $begingroup$
    You’re welcome, glad I could help.
    $endgroup$
    – Jake O
    Mar 14 at 14:42
















$begingroup$
The latter one. It looks like there is a mistake in the answers, then. Thank you very much for solving it.
$endgroup$
– Marcin
Mar 14 at 14:24




$begingroup$
The latter one. It looks like there is a mistake in the answers, then. Thank you very much for solving it.
$endgroup$
– Marcin
Mar 14 at 14:24












$begingroup$
You’re welcome, glad I could help.
$endgroup$
– Jake O
Mar 14 at 14:42




$begingroup$
You’re welcome, glad I could help.
$endgroup$
– Jake O
Mar 14 at 14:42



Popular posts from this blog

Magento 2 - Add success message with knockout Planned maintenance scheduled April 23, 2019 at 23:30 UTC (7:30pm US/Eastern) Announcing the arrival of Valued Associate #679: Cesar Manara Unicorn Meta Zoo #1: Why another podcast?Success / Error message on ajax request$.widget is not a function when loading a homepage after add custom jQuery on custom themeHow can bind jQuery to current document in Magento 2 When template load by ajaxRedirect page using plugin in Magento 2Magento 2 - Update quantity and totals of cart page without page reload?Magento 2: Quote data not loaded on knockout checkoutMagento 2 : I need to change add to cart success message after adding product into cart through pluginMagento 2.2.5 How to add additional products to cart from new checkout step?Magento 2 Add error/success message with knockoutCan't validate Post Code on checkout page

Fil:Tokke komm.svg

Where did Arya get these scars? Unicorn Meta Zoo #1: Why another podcast? Announcing the arrival of Valued Associate #679: Cesar Manara Favourite questions and answers from the 1st quarter of 2019Why did Arya refuse to end it?Has the pronunciation of Arya Stark's name changed?Has Arya forgiven people?Why did Arya Stark lose her vision?Why can Arya still use the faces?Has the Narrow Sea become narrower?Does Arya Stark know how to make poisons outside of the House of Black and White?Why did Nymeria leave Arya?Why did Arya not kill the Lannister soldiers she encountered in the Riverlands?What is the current canonical age of Sansa, Bran and Arya Stark?